Sets in R Note

You might also like

Download as pdf or txt
Download as pdf or txt
You are on page 1of 25

Sets in ℝ

Open Sets
1) Define Neighbourhood of a point with examples.
Ans: Let c ∈ ℝ. A subset S ⊆ ℝ is said to be a neighbourhood of c if there exists an open
interval (a, b) such that c ∈ (a,b) ⊆ S, where a,b ∈ ℝ with a < b.
Example: If (a,b) be an open interval, where a,b ∈ ℝ with a < b.
Then (a,b) is a neighbourhood of each point of (a,b).
𝟏
2) What is meant by neighbourhood of a point? Check whether the set { 𝒏 : 𝒏∈ℕ } ∪ {0} is a
neighbourhood of 0 or not. [C.U-14, 18]
Ans : 1st part :- see Q.No. (1).
1
2nd part :- Let S = { 𝑛 : 𝑛∈ℕ} ∪ {0}

There exists no (a,b) such that 0 ∈ (a,b)⊆ S were a,b ∈ ℝ with a < b.
∴ S is not a neighbourhood of ‘0’
3) Define 𝜺-neighbourhood of a point with examples.
Ans: →Let a ∈ ℝ and 𝜀 > 0. Then 𝜀 −neighbourhood of a = N(a, 𝜀) = (a−𝜀, a+𝜀)− open
interval.
Example: i) N(2,3) = (2−3,2+3) =(−1,5);
( | )
ii) N(3,2) = (3−2, 3+2) = (1,5) a−ε a a+ε
4) Define interior point of a set S ⊆ ℝ. [C.U.-03]
Ans :→Let S ⊆ ℝ and a∈S. Then a is called an interior point of S iff ∃𝜀 > 0 such that N(a,𝜀) ⊆S.
Example: let a,b, c ∈ ℝ
Here c is an interior point of [a,b] where a < c < b.
But a,b are not.
5) Does their exist two non-empty disjoint sets of real numbers having the same set of interior
points? Justify your answer [C.U.-14]
Ans : → Let A = ℕ, B = – ℕ
Here A∩B = ∅, so they are disjoint sets.
But int(A) = int(B) = ∅
6) Prove or disprove: Every bounded infinite subset of ℝ has an interior point. [C.U-17]
Ans : → The given statement is false.
1
Counterexample : Let S = { 𝑛 : 𝑛∈ℕ}.
Here S is bounded as S ⊆ [−1,1] and infinite
But S has no interior point.
7) Define interior of a set S ⊆ ℝ. [C.U.-03]

Ans :→ The set of all interior points of a set S ⊆ ℝ is called the interior of S and it is denoted
by int (S) or S o i.e., 𝑎 ∈ int (S) iff ∃ 𝜀 > 0 such that N(a,𝜀) ⊆ S.
Example : (i) If S = ℕ or ℤ or ℚ, then S o = ∅.
(ii) If S = (a,b) or [a,b) or (a,b] or [a,b], then S o = (a,b) where a,b ∈ ℝ with a < b.
8) Define open set in ℝ. [C.U.-01,03]
Ans :→ A set S ⊆ ℝ is called an open set iff every point of S is an interior point of S i.e., A set
S⊆ ℝ is called an open set in ℝ iff ∀𝑎∈S , ∃𝜀 > 0 such that N(a,𝜀) ⊆ S.
Example : (i) If S = ℕ or ℤ or ℚ, then S is not open.
(ii) If S = (a,b), then S is open but S = [a,b) or (a,b] or [a,b], then S is not open, where a,b ∈ ℝ
with a<b.
9) Show that the set S = {𝒙 ∈ ℝ: |𝒙 −1|+| 𝒙 −2|<3} is an open Set. [C.U-06]
Ans :→ Let 𝒙 ∈ S
If 𝑥 < 1, 𝑡ℎ𝑒𝑛 |𝑥 − 1| + | 𝑥 − 2| = (1 − 𝑥) + (2 − 𝑥) = 3 – 2𝑥 < 3 ⇒ −2𝑥 < 0 ⇒ 𝑥 > 0
𝐼𝑓 𝑥 ≥ 2, 𝑡ℎ𝑒𝑛 |𝑥 − 1| + | 𝑥 − 2| = (𝑥 − 1) + (𝑥 − 2) = 2𝑥 – 3 < 3 ⇒ 𝑥 < 3
𝑖𝑓 1 ≤ 𝑥 < 2, 𝑡ℎ𝑒𝑛 |𝑥 − 1| + | 𝑥 − 2| = (𝑥 − 1) + (2 − 𝑥) = 1 < 3
which is always true.
𝑥 −0 3−𝑥
So S = (0,3) ( )
Now (0,3) is open. Because, ∀𝑥 ∈ (0,3), 0 𝑥 3

we can choose 𝜀 = min { 𝑥, 3−𝑥} > 0 such that N(𝑥, 𝜀) ⊆ (0,3),


As required.
10)Let S be a subset of ℝ. Then show that int(S) is an open set. [C.U.-05]
Ans :→ To show : int(S) is open
i.e., ∀𝑎 ∈ int(S), ∃ 𝜀 > 0 Such that N(𝑎, 𝜀) ⊆ int(S).
Casa 1− If int(S) = ∅ , it is vacuously true that ø is open.
Case 2− If int(S) ≠ ∅ , then take any 𝑎 ∈ int (S)
∵ 𝑎 ∈ int(S), then a is an interior point of S.
i.e., ∃ 𝜀 > 0 such that N(𝑎, 𝜀) ⊆ S.
Choose this 𝜀 > 0, shall show : N(𝑎, 𝜀) ⊆ int(S).
i.e., ∀𝑥 ∈ N(𝑎, 𝜀) ⇒ 𝑥 ∈ int (S)
Take any 𝑥 ∈ N(𝑎, 𝜀)
Since N(𝑎, 𝜀) is open, then 𝑥 is an interior point of N(𝑎, 𝜀),
Therefore ∃ δ > 0 such that N(𝑥,δ) ⊆ N(𝑎, 𝜀) ⊆ S
So, 𝑥 is an interior point of S, i.e., 𝑥 ∈ int(S)
Since 𝑥 ∈ N(𝑎, 𝜀) is arbitrary, so N (𝑎, 𝜀) ⊆ int(S)
Since 𝑎 ∈ int(S) is arbitrary, so int(S) is open obviously.

11)Let S ⊆ ℝ. Then show that int(S) is the largest open set contained in S.

Ans :→ 1st part − you have to show int(S) is an open set as earlier.
2nd part − By definition, int(S) ⊆ S.
To show: For any open set I ⊆ S in ℝ, I ⊆ int(S).
i.e., ∀ a ∈ I ⇒ a ∈ int(S).
Take any a ∈ I
∵ I is open, ∃ ε > 0 such that N(a, ε) ⊆ I ⊆ S
∴ a is an interior point of S, i.e., a ∈ int(S)
Hence I ⊆ int(S),
As required.
12)Let A, B be subsets of ℝ . Then prove that A ⊆ B ⇒ 𝐀𝐨 ⊆ 𝐁 𝐨 [where 𝐀𝐨 = int(A)]

Proof : Given that A ⊆ B

To show: Ao ⊆ Bo

Let any a ∈ Ao , then a is an interior point of A.

∴ ∃ 𝜀 > 0 such that N(a, 𝜀) ⊆ A ⊆ B (given)

So a is an interior point of B.

∴ a ∈ Bo

Since a is arbitrary, Ao ⊆ Bo (proved)

13)Let A, B be subsets of ℝ. Then prove that 𝐀𝐨 ∪ 𝐁 𝐨 ⊆ (A∪B)o

Proof :- Since for any two sets S,T ⊆ ℝ, S⊆ T ⇒ S o ⊆ T o


Then A ⊆ A∪B ⇒ Ao ⊆ (A∪B)o ------(i)
and B ⊆ A∪B ⇒ Bo ⊆ (A∪B)o ------(ii)
From (i) & (ii), Ao ∪ Bo ⊆ (A∪B)o (proved)
14)Let A, B be subsets of ℝ. Then prove that (A⋂B)o = 𝐀𝐨 ⋂𝐁 𝐨
Proof : Since for any two sets S,T ⊆ ℝ, S⊆ T ⇒ S o ⊆ T o

Then A⋂B ⊆ A ⇒ (A⋂B)o ⊆ Ao ------(i)

and A⋂B ⊆ B ⇒ (A⋂B)o ⊆ Bo ------(ii)


So from (i) & (ii), (A⋂B)o ⊆ Ao ⋂Bo ------(iii)

Now we shall prove that, Ao ⋂Bo ⊆ (A⋂B)o

Let any a∈ Ao ⋂Bo ⇒ a∈ Ao and a∈ Bo

Then a is an interior of A as well as B.

Hence, ∃𝜀1, 𝜀2 > 0 such that N(a, 𝜀1 ) ⊆ A and N (a, 𝜀2 ) ⊆ B

Let 𝜀 = min { 𝜀1 , 𝜀2 } > 0

Then N(a, 𝜀) ⊆ N(a,𝜀1 ) ⊆ A and N(a,𝜀) ⊆ N (a, 𝜀2 ) ⊆ B ⇒ N(a,𝜀) ⊆ A⋂B.

So, ∃ 𝜀 >0 such that N(a, 𝜀) ⊆ A⋂B

∴ a is an interior point of A⋂B

i.e., a∈(A⋂B)o

Since a is arbitrary, Ao ⋂Bo ⊆ (A⋂B)o ------ (iv)

From (iii) & (iv), (A⋂B)o = Ao ⋂Bo (proved)

15)Show that every non-empty open set can be expressed as a union of open intervals. [C.U.-
13-]
Ans :→ Let A ≠ ∅ be an open set in ℝ.
Then ∀𝑎 ∈A, ∃𝜀𝑎 > 0 Such that N(a, 𝜀𝑎 ) ⊆ A. ------- (i)
To show: A = ∪ N (a, 𝜀𝑎 ). Let B= ∪ N (a, 𝜀𝑎 )
𝑎∈𝐴 𝑎∈𝐴

Since for any 𝑎 ∈ A, N(a, 𝜀𝑎 ) ⊆ A. [from (i)]


Therefore B ⊆ A -------(ii)
Also for any 𝑎 ∈A, 𝑎 ∈ N(a, 𝜀𝑎 ) ⊆ B
Therefore A ⊆ B -------(iii)
From (ii) and (iii), A = B,
As required.
16)Prove that the set S is an open set, where S = { 𝒙 ∈ ℝ : Sinx ≠ 0 }
Ans :→
−2π. . . .
. −π π 2π
Here S = { 𝑥∈ℝ : Sinx ≠ 0 }
= ------ ∪ (−2π, − π) ∪(−π,0) ∪ (0, π) ∪ (π,2 π) ∪ ------
= ∪ (nπ, nπ+π)
𝑛∈ℤ
Since every open interval is an open set and the union of an arbitrary collection of open sets
in ℝ is an open set.
So S is an open set.
17) Prove that the union of a finite number of open Sets in ℝ is an open set.
Ans :→ Let G1 ,G2 , ----- , G𝑛 be finitely many open sets for some n∈ℕ
Let G = G1 ∪G2 ∪----- ∪G𝑛
To show: ∀𝑎 ∈ G, ∃ 𝜀 > 0 such that N(a,ε) ⊆ G
Let any 𝑥 ∈ G ⇒ 𝑥 ∈ G𝑖 for some i ∈ {1,2,-----,𝑛}
Since G𝑖 is an open set, then 𝑥 is an interior point of G𝑖 .
Hence, ∃ε𝑖 > 0 such that N(𝑥, ε𝑖 ) ⊆ G𝑖 ⊆ G.
This shows that 𝑥 is an interior point of G. since 𝑥 is arbitrary, G is an open set (proved).
18)Show that the intersection of a finite number of open sets is an open set. With the help of a
suitable example show that the intersection of an infinite collection of open Sets need not
always be an open set [C.U.-99,05, 11,].

Ans :→ 1st part -


Let G1 ,G2 , ----- , G𝑛 be finitely many open sets for Some 𝑛∈ℕ.
Let G = G1 ∩G2 ∩ ---- ∩G𝑛
To show: ∀𝑎 ∈ G, ∃ ε > 0 such that N(a,ε) ⊆ G.
If G = ∅, then it is vacuously true otherwise take any 𝑎 ∈ G ⇒ 𝑎 ∈ G𝑖 for all i∈ {1,-----,𝑛}
Since each G𝑖 is open, then ∃ ε𝑖 > 0 such that N(a,ε𝑖 ) ⊆ G𝑖 for all i∈ {1,-----,𝑛}
We can choose ε = min {ε1 , ε2 ,-----, ε𝑛 } > 0
Then N(a,ε) ⊆ N(a, ε𝑖 ) ⊆ G𝑖 , ∀i∈ {1, 2,-----,𝑛}
So N(a,ε) ⊆ G1 ∩G2 ∩ ----- ∩G𝑛 = G,
As required
2nd part-
The intersection of an infinite collection of open sets need not be open.
1 1
Example → take I𝑛 =(− 𝑛 , 𝑛 ) for all 𝑛∈ℕ.

Then ⋃∞
𝑛=1 I𝑛 = {0} which is not open.

CLOSED SET
19)Define closed set with examples.
Ans :→ A set F ⊆ ℝ is called closed in ℝ if F c is open in ℝ

Example – (1) [a,b], where a < b is closed since [a,b]c = (− ∞, a) ∪ (b, ∞) is open.

(2) ∅ is closed, since ∅c = ℝ is open


20)Prove that every finite subset of ℝ is a closed set. [C.U.-97]

Ans :→ Let F = {𝑎1 , 𝑎2 , 𝑎3 ,------, 𝑎𝑛 } be any finite set, where 𝑎1 < 𝑎2 < ------ < 𝑎𝑛 .

To show : F is closed, it is enough to show that F c is open.


Now F c = (− ∞, 𝑎1 ) ∪ (𝑎1 , 𝑎2 ) ∪ ------ ∪ (𝑎𝑛−1 , 𝑎𝑛 ) ∪ (𝑎𝑛 , ∞)
We know that every open interval is open set and the union of a finite number of open sets is
an open set.
So F c is open, then F is closed.

21)Give illustrations of two non-empty open sets 𝐆𝟏 , 𝐆𝟐 and two non-empty closed sets 𝐇𝟏 ,
𝐇𝟐 in ℝ where 𝐆𝒊 ∩ 𝐇𝒊 = ∅, i =1, 2 such that 𝐆𝟏 ∪ 𝐇𝟏 is an open set and 𝐆𝟐 ∪ 𝐇𝟐 is a closed
set in ℝ. [C.U-05]
Ans :→ Let G1 = (0,1) ∪ (2,3) and H1 = [1,2]
So, G1 ∩ H1 = ∅ and G1 ∪H1 = (0,3)
which is open
Also we let G2 = (1, 2) and H2 = [0,1] ∪ [2,3]
Then G2 ∩H2 = ∅ and G2 ∪H2 = [0,3]
which is closed.
22)Prove that the intersection of two closed sets is a closed Set. Hence show that if G ⊆ ℝ is an
open set and F ⊆ ℝ is a closed set, then F\G is a closed set [C.U.-12]
Proof: part 1 : → Let F1 , F2 be two closed sets
To show : F = F1 ∩ F2 is closed.
i.e., F c = F1c ∪ F2c is open
Let 𝑥 ∈ F c ⇒ 𝑥 ∈ F1c ∪ F2c ⇒ 𝑥 ∈ F1c or 𝑥 ∈ F2c
Let 𝑥 ∈ F1c
Since F1c is open, then 𝑥 is an interior point of F1c
⇒ ∃ε > 0 such that N(𝑥, ε) ⊆ F1c ⊆ F1c ∪ F2c = F c
So, 𝑥 is an interior point of F c
Since 𝑥 is arbitrary, F c is open.
If 𝑥 ∈ F2c , Similarly, F c is open.
As required.
Part 2 :→ Given that G is open and F is closed.
To show : F \ G is closed
Now F \ G = F ∩ Gc is closed
Since F, Gc are closed and the intersection of two closed sets is closed.
23)Prove that the union of a finite number of closed sets is closed. Justify for infinitely closed
sets.[C.U.-07]
Ans : → 1st part : -
Let F1 , F2 , ----- , F𝑛 be closed.
To show : F = F1 ∪F2 ∪--- ∪F𝑛 is closed
i.e., F 𝑐 = F1c ∩ F2c ∩ --- ∩Fnc (By De Morgan's Law) is open
Let G𝑖 = F𝑖c and G = F c
Given that F𝑖c is open for 𝑖 =1,2,-----, 𝑛 .
To show : G is open i.e., ∀ 𝑥 ∈ G, ∃ε > 0 such that N(𝑥, ε) ⊆G
If G = ∅, then it is vacuously true that ∅ is open.
If G ≠ ∅, then we take any 𝑥 ∈ G
⇒ 𝑥 ∈ G𝑖 , for all i ∈ {1,2,-----,𝑛}.
Since each G𝑖 is open, then 𝑥 is an interior point of G𝑖 .
i.e., ∃ε𝑖 >0 such that N(𝑥, ε𝑖 )⊆ G𝑖 , for all i∈ {1,-----,𝑛}.
We choose ε = min {ε1 , ε2 ,-----, ε𝑛 } > 0
Then N(𝑥,ε) ⊆ N (𝑥, ε𝑖 ) ⊆ G𝑖 for all i∈ { 1,-----, 𝑛}
So N(𝑥,ε) ⊆ G,
As required.
2nd part :-
The union of an infinite number of closed sets in ℝ is not necessarily a closed set.
1 1
Example - Take F𝑛 =[ 𝑛, 3− ] for all 𝑛∈ℕ
𝑛

Then ∪ F𝑛 = (0,3) which is not closed.


𝑛∈ℕ

24)Let G be an open set in ℝ and S be a non-empty finite Subset of ℝ. Prove that G \ S is an


open set.
Ans : → Since every non-empty finite set is closed in ℝ.
Then S is closed and S c is open.
Also given G is an open set.
Then G \ S = G ∩ S c is open, since the Union of two open sets is open.
LIMIT POINTS
25)Define limit point of a subset of real numbers.
Ans : → Let S be a subset of ℝ and 𝑥∈ℝ.
Then 𝑥 is called a limit point of S if ∀ε > 0, N′(𝑥,ε)∩S ≠ ∅
Example :
1
(i) S = {𝑛 : 𝑛∈ℕ}. Then '0' is a limit point of S, since ∀ε > 0, N′(0, ε)∩S ≠ ∅

26)Let 𝒙∈ℝ be an interior point of a set A and a limit point of a set B. If A\B ≠ ∅, can you
conclude that 𝒙 is an interior point of A\B? Give reasons in a support of your answer [C.U.-
14].

Ans: → This statement is false


1
Counterexample : Let A = (0,1) and B=( 2, 2)
1
Here 2 is an interior point of A and a limit point of B
1 1
But 2 is not an interior point of A\B = (0, 2].

27)Define Isolated point of a set of ℝ.


Ans : → Let S be a subset of ℝ and 𝑥∈S. Then 𝑥 is called an isolated point of S if 𝑥 is not a limit
point of S. i.e., ∃ε > 0, N′(𝑥,ε)∩S = ∅
1
Example : S = {𝑛 : 𝑛 ∈ ℕ}. Every point of S is an isolated point of S.

28)Prove that every neighbourhood of a limit point 𝒙 of S contains infinite number of points of
S. [C.U.-94]
Ans : → Here 𝑥 is a limit point of S and N(𝑥, ε) is any neighbourhood of 𝑥, where ε > 0.
If possible we let N′(𝑥, ε) contains finite number of points 𝑥1 , 𝑥2 ,-----, 𝑥𝑛 of S.
Let 𝛿𝑖 =|𝑥 − 𝑥𝑖 | > 0 , ∀i∈ {1,2,-----,𝑛} Contains no point of S
𝑥 𝑥 𝑥 𝑥
and again we let 𝛿 = min { 𝛿1 , 𝛿2 ,-----, 𝛿𝑛 } > 0 ( . . 3. ( | ) .4 . .6 . .8 )
𝑥 − 𝜀 𝑥1 𝑥2 𝑥 𝑥5 𝑥7 𝑥 + 𝜀
Now, clearly N′(𝑥, 𝛿) ∩S = ∅
Therefore a deleted neighbourhood of 𝑥 contains no point of S,
Then 𝑥 is not a limit point of S, A contradiction.
Therefore every neighbourhood of 𝑥 contains infinitely many points of S.
29)If A be a non-empty bounded subset of ℝ. show that A′ ∩ (b,∞) = ∅, where b = Sup A and
A′ is the derived set of A [C.U.-14]

Ans :- If possible we let , A′ ∩ (b,∞) ≠ ∅


Let any 𝑥 ∈ A′∩ (b,∞)
⇒ 𝑥 ∈ A′ and 𝑥 ∈ (b,∞)
Now 𝑥 ∈ A′ ⇒ 𝑥 is a limit point of A
⇒ ∀ ε > 0 ,N(𝑥, ε) contains infinite number of points of A. ------ (i)
and 𝑥 ∈ (b,∞) ⇒ 𝑥 is an interior point of (b,∞)
⇒ ∃ δ > 0, N(𝑥,δ) ⊆ (b,∞)
If we choose ε = δ, then from (i),
N(𝑥,δ) contains infinite number of points of A
Since N(𝑥,δ) ⊆ (b,∞), then (b,∞) contains infinite number of points of A.
So, Sup(A) > b , a contradiction
Hence our assumption is incorrect.
∴ A′∩ (b,∞) = ∅.

30)State and prove Bolzano-Weierstrass theorem for set in ℝ. [C.U -97,02,06,09,11]

Ans : → Statement : Every infinite bounded subset of ℝ has at least one limit point in ℝ.
Proof : Let S be any bounded and infinite set in ℝ. Since S is a non-empty bounded set, then
Sup(S) and inf(S) both exist in ℝ.
Let Sup(S) =M, inf(S)= 𝑚.

Consider a set H = {𝑥∈ℝ: 𝑥 exceeds at most finite number of elements of S}.


i.e., 𝑥∈H iff either 𝑥 is greater than no element of S or 𝑥 is greater than only a finite number
of elements of S.
Since ∀𝑦∈S, y ≥ 𝑚 . ∴ 𝑚∈H as 𝑚 is not greater than any element of S.
∴ H is non-empty
Also the set H is bounded above by M, since M is greater than an infinite number of elements
of S. ∴ By completeness axiom, H has a Supremum in ℝ
Let Sup H = ∝. Then for any ε > 0, ∃β∈H Such that β > ∝ − ε
β
( | )
∝−ε ∝ ∝+ε
Since β∈H, therefore β exceeds at most finite number of elements of S
Since ∝ −ε < β, therefore (∝-ε) exceeds at most finite number of elements of S and (∝ + ε)
is greater than an infinite number of elements of S as ∝ + ε ∉ H .
So the interval (∝−ε, ∝+ε) contains an infinite number of elements of S.
Therefore, ∀ ε > 0, N(∝, ε) ∩S is infinite
∴ ∝ is a limit point of S.
31)Let A be a bounded infinite subset of ℝ.
Let B = {𝒙∈ℝ : 𝒙 is greater than infinitely many points of A}. Show that inf B is a limit point of
A.
Ans : → Same as before.
32)Can you apply the Bolzano - Weierstrass theorem for the set {1, 2, 3,-----} ? Justify your
answer. [C.U.-10]
Ans : → No, we cannot, because the set is not bounded.
𝟏
33)Verify the Bolzano-Weierstrass theorem for the set {1 + 𝒏: 𝒏∈ℕ}. [C.U.-99]
1
Ans : → Let S = {1 + 𝑛: 𝑛∈ℕ}

The set S is infinite, since ℕ is infinite


1
∀𝑛∈ℕ, 1 < 1 + 𝑛 ≤ 2

Therefore S is bounded
So, by Bolzano-Weierstrass theorem, S must have a limit point in ℝ.
To show: 1 is a limit point of S.
i.e., ∀ε > 0, N′(1,ε) ∩S ≠ ∅.
Take any ε > 0
1
By Archimedean property, ∃𝑛∈ℕ , 0 < < 𝜀
𝑛
1
⇒1<1+𝑛<1+ε
1
⇒ 1 + 𝑛 ∈ (1, 1+ε ) ⊆ N′(1,ε)
∴ N′(1, ε) ∩S ≠ ∅,
As required.

34)Prove or disprove. If S is an infinite bounded above Subset of ℝ, then S has a limit point.
[C.U. -17]

Ans : → The given statement is false.


Counterexample : Let S = − ℕ
Here S is an infinite bounded above by (−1).
But it has no limit point in ℝ.

35)Prove or disprove: There is no bounded infinite set whose points are isolated. [C.U.18]
Ans : → This statement is false.
1
Counterexample : Let S = {𝑛 : 𝑛∈ℕ}

Here S is a bounded as S ⊆ [−1,1] and infinits Set.


It has only one limit point which is 0, but 0 ∉ S
So every point of S is an isolated point.
36)Define derived set with examples.
Ans : → Let S ⊆ ℝ. The set of all limit points of S is called the derived set of S and it is denoted
by S′ . Example : -
(1) Let S be a finite set. Then S′ = ∅
(2) Let S = ℕ or ℤ. Then S′ = ∅
(3) Let S = ℚ. Then S′ = ℝ
𝟑
37)Let A ⊆ ℝ be defined by A = {2 + 𝒓: r ∈ (0,1] ∩ ℚ }, Find the derived set of A. (C.U-15]

Ans : → Given that r is a rational number


1 3 3
and 0 < r ≤ 1 ⇒ 1 ≤ 𝑟 < ∞ ⇒ 3 ≤ 𝑟 < ∞ ⇒ 5 ≤ 2+ 𝑟 < ∞
So, A = [5,∞) ∩ ℚ .
Since ℚ is dense in ℝ, then A is also dense in [5,∞).
∴ Every point of [5,∞) is a limit point of A.
Therefore A′ = the derived set of A = [5,∞).

38)Let A,B be subsets of ℝ and A ⊆ B. Then prove that A' ⊆ B'.


Ans : → If A′ = ∅ , then obviously A′ ⊆ B′.
If A′ ≠ ∅, then take any a∈A′
⇒ a is a limit point of A.
∴ ∀𝜀 > 0, N′(a,ε) contains a point b of A.
Since A⊆B, b∈A ⇒ b∈B.
∴ ∀ε > 0, N′(a,ε) contains a point b of B.
∴ a is a limit point of B ⇒ a∈B′
Since a is arbitrary, A′⊆ B′.
39)Let S ⊆ ℝ .Then show that (S′)′ ⊆ S′ or, prove that the derived set of a subset of real
numbers is a closed set. [C.U -01,03,04,09]

Ans : → Let S ⊆ ℝ
To show: S′ is closed in ℝ.
i.e., (S′)c is open in ℝ
i.e., ∀𝑥∈(S′)c , ∃ε > 0 such that N(𝑥,ε) ⊆ (S′)c ,
If (S′)c = ∅, Then it is vacuously true
If (S′)c ≠ ∅, then for any 𝑥∈(S′)c ⇒ 𝑥 ∉ S′
Then 𝑥 is not a limit point of S
i.e., ∃ε> 0, N′(𝑥,ε) ∩S = ∅
i.e., ∃ε> 0, N(𝑥,ε) ⊆ S c
Now we show N(𝑥,ε) ⊆ (S′)c
We take any 𝑦∈N(𝑥,ε)
Since N(𝑥,ε) is open, then ∃δ > 0 such that N (y,δ) ⊆ N(𝑥,ε) ⊆ S c
i.e., ∃ δ > 0, N′(𝑦,δ) ∩S = ∅
N(𝑥,ε)← →N(𝑥,ε)
So, 𝑦 is not a limit point of S ⇒ 𝑦 ∉ S′ | |
← S →
⇒ 𝑦 ∈ (S′)c
S N(𝑥,ε)
Since y is arbitrary, N(x, 𝜀) ⊆ (S′)c .
Since 𝑥 is arbitrary, (S′)c is open in ℝ
i.e., S′ is closed in ℝ (proved). Venn diagram

Alternative proof. - Let S ⊆ R


To show S′ is closed in ℝ i.e., (S′)′ ⊆ S′

Case 1 : If (S′)′ = ∅. Then (S′)′ ⊆ S′,

As required

Case 2 : If (S′)′ ≠ ∅. Then let a ∈ (S′)′

∴ a is a limit point of S′

i.e., ∀ ε > 0, N′(a,ε) contains a point b of S′

∴ b∈S′, so b is a limit point of S. ( | | )


a−ε b a a+ε
i.e., ∀ δ > 0, N(b,δ) contains infinitely many points of S. ε > |a – b|
If we chose δ = ε – |a−b| > 0

Then N(b, δ) contains infinitely many points of S and N(b,δ) ⊆ N(a,ε)

So N(a,ε) contains infinitely many points of S .

Therefore a is a limit point of S. i.e., a∈S′

Since a is arbitrary, then (S′)′ ⊆ S′,

As required.

40)Let A,B ⊆ R. Then prove that (A∩B)′ ⊆ A′ ∩ B′


Proof: Since S ⇒ T → S′ ⊆ T′ , where S, T ⊆ ℝ
∴ (A∩B) ⊆ A ⇒ (A∩B)′ ⊆ A′ - (i)
and (A∩B) ⊆ B ⇒ (A∩B)′ ⊆ B′ - (ii)
From (i) & (ii), (A∩B)′ ⊆ A′∩B′ (proved)
41)Let A, B ⊆ ℝ. Then prove that (A∪B)′ = A′∪B′
Proof : Since S ⊆ T ⇒ S′ ⊆ T′, where S, T ⊆ ℝ
∴ A ⊆ A∪B ⇒ A′ ⊆ (A∪B)′
and B ⊆ A∪B ⇒ B′ ⊆ (A∪B)′
So, A′∪B′ ⊆ (A∪B)′ ------(i).
Now, to show: (A∪B)' ⊆ A'∪B'
Let any a ∉ A′∪B′ ⇒ a∉ A′ and a ∉ B′
∴ a is not a limit point of A as well as B.
So, ∃ 𝜀1 , 𝜀2 > 0 such that N′(a, 𝜀1 ) ∩A = ∅ and N′(a, 𝜀2 ) ∩B = ∅
Let ε = min { 𝜀1 , 𝜀2 }. Then ∃ ε > 0, N′(a,ε) ∩ A = N′(a,ε) ∩B = ∅
Therefore N′(a,ε) ∩ (A∪B) = (N′(a,ε) ∩ A) ∪ (N′(a,ε) ∩B)
=∅∪∅=∅
So, a is not a limit point of A∪B.
∴ a ∉ (A∪B)′
Hence a ∉ A′∪B′ ⇒ a ∉ (A∪B)′
Contrapositively, a ∈ (A∪B)′ ⇒ a ∈ A′∪B′
Since a is arbitrary, (A∪B)′ ⊆ A′∪B′ ------(ii)
From (i) & (ii), (A∪B)′ = A′∪B′ (proved)

42)Let A be a subset of ℝ. Prove that an interior point of A is a limit point of A.


Proof : Let a is an interior point of A
Then ∃ δ > 0 such that N(a,δ) ⊆ A
To show : a is a limit point of A.
i.e., ∀ ε > 0, N′(a,ε) ∩ A ≠ ∅
Take any ε > 0
Case 1 : -
If 0 < ε < δ. Then N(a, ε) ⊆ N(a,δ) ⊆ A
∴ N′(a, ε) ∩A = N′(a, ε) ≠ ∅
Case 2 : - If ε ≥ δ > 0,
Then N(a,δ) ⊆ N(a, ε) and N(a,δ) ⊆ A
⇒ N (a,δ) ⊆ N(a, ε)∩A
Hence clearly, N′(a, ε) ∩ A ≠ ∅.
∴ a is a limit point of A (proved).
43)If A, B are any two subsets of ℝ that differ by only finitely many elements, then A′ = B′.
44)For any A ⊆ ℝ and a, b∈ℝ with a ≠ 0. Then (aA + b)′ = aA′ + b
𝟏
45)Find the derived set of S = { 𝒏 : 𝒏∈ℕ } [C.U.- 08, 10,-----]
1
Ans : → By the Archimedean property of ℝ, ∀ε > 0, ∃𝑛∈ℕ such that 0 < 𝑛 < ε
1 1
i.e., ∀ε > 0, ∃𝑛∈ℕ such that 𝑛 ∈(0,ε) ⇒ ∈ N′(0,ε)
𝑛

∴ ∀ε > 0, N′(0,ε) ∩ S ≠ ∅. So 0 is a limit point of S. So 0 is a limit point of S.


Now To show: if a ≠ 0, then a is not a limit point of S.
i.e., ∀ a ≠ 0, ∃ ε > 0, N(a, ε) ∩S is a finite set.
Take any a ≠ 0.
|𝑎−0| |𝑎|
We choose ε = 2
= 2
>0
1
Then by Archimedean property, ∃k∈ℕ such that 𝑘 < ε
1 1
So, ∀ 𝑛 ≥ k , 0 < 𝑛 ≤ 𝑘 < ε Infinite finite elements
of S
1 | ( | )
i.e., ∀ 𝑛 ≥ k , 𝑛 ∉ N(a, ε) |𝑎| |𝑎|
0 a− a a+
2 2
so N(a, ε) ∩ S is finite. ∝+ε
As required.
∴ The derived set of S = S′ = { 0 }

𝟏
46)Find the derived set of S = {(−1)𝒎 + 𝒏 : m, 𝒏∈ℕ} [C.U.01]
1
Ans : → Here S = {(−1)𝑚 + : 𝑚, 𝑛∈ℕ}
𝑛
1 1
= {−1 + : 𝑛∈ℕ} ∪ {1+ 𝑛 ∶ 𝑛∈ℕ}
𝑛
1 1
Let S1 = {−1 + : 𝑛∈ℕ}, S2 = {1+ : 𝑛∈ℕ}
𝑛 𝑛
= −1 + A =1+A
1
Where A = { 𝑛 : 𝑛∈ℕ}

From standard result we know that A' = { 0 }


By property of derived sets,
S1′ = − 1 + A′ = − 1 + {0} = {−1}
S2 ′ = 1 + A′ = 1 + {0} = {1}
Again by the property of derived sets for finite union, S′= (S1∪S2 )′ = S1 ′∪ S2 ′= { −1, 1}(Ans)
𝒏−𝟏
47)Given E = { 𝒏+𝟏 : 𝒏ϵℕ} U { 2,3}

Find the set of all accumulation points and the set of isolated points of E. [C.U. -99,]
𝑛−1
Ans : → Here E = { 𝑛+1 : 𝑛∈ℕ} ∪ { 2,3}
2
={1− : 𝑛∈ℕ} ∪ { 2,3}
𝑛+1

= (1− 2A) ∪ {2,3}


1
Where A ={ 𝑛+1 : 𝑛∈ℕ}
1
Again we let B = { 𝑛 : 𝑛∈ℕ}. Then B = A ∪ {1}

⇒ B′ = A′∪ ∅

⇒ A′ = { 0 }, by standard result.

Also {2,3}′ = ∅, since finite set.

So E′ = (1− 2A′) ∪ {2,3}′

= (1−2 {0}) ∪ ∅

={1}
𝟏 𝟏
48)Find the derived set of E = { 𝒎 + 𝒏 : 𝒎, 𝒏∈ℕ}
1 1
Ans : → Here E = { 𝑚 + 𝑛 : 𝑚, 𝑛∈ℕ }
1 1
= { 𝑚 + 𝑛 : 𝑚 ≥ 𝑛 where 𝑚, 𝑛∈ℕ }
1 1
= ∪ { 𝑚 + 𝑛 : 𝑚 ≥ n, 𝑚∈ ℕ }
𝑛∈ℕ
1 1
= ∪ E𝑛 , where E𝑛 = { 𝑚 + 𝑛 : 𝑚 ≥ 𝑛, 𝑚∈ ℕ}
𝑛∈ℕ
1 1 1 1
Now, ∀𝑛∈ℕ, E𝑛 = 𝑛 + { 𝑚 : 𝑚 ≥ 𝑛, 𝑚∈ ℕ} = 𝑛 + S𝑛 , where S𝑛 = { 𝑚 : 𝑚 ≥ 𝑛, 𝑚∈ ℕ}
1
Let S = { 𝑛 : 𝑛∈ℕ}

Then S𝑛 and S differ by finitely many elements.


So S𝑛 ′ = S′ = {0} .
1 1 1
So E𝑛′ = 𝑛 + S𝑛′ = 𝑛 + {0} = { 𝑛 }

Since E𝑛 ⊆ E, then E𝑛′ ⊆ E′, ⇒ N∈ℕ


1
Thus ∀𝑛∈ℕ, 𝑛 ∈E′
1 1 2
Also ∀𝑛∈ℕ, 𝑛 + 𝑛 = 𝑛 ∈E
∴ 2 S ⊆ E ⇒ 2S′ ⊆ E′ ⇒ 2{0} ⊆ E′
⇒ {0} ⊆ E′
1
We see that, A⊆ E′ where A = { 0 } ∪ {𝑛 : 𝑛∈ℕ}
Fix any p∈ℕ,
Then E = E1∪E2 ∪ ----- ∪E𝑝−1 ∪E𝑝 ∪E𝑝+1 ∪-----
2
Let F𝑝 = E𝑝 ∪E𝑝+1 ∪---- ⊆ (0, 𝑝 )
2
⇒ F𝑝′ ⊆ [ 0, 𝑝 ]
1 1 2
So, E′= E1′ ∪ ----- ∪E𝑝−1

∪F𝑝′ ⊆ {1, 2 , ----- , 𝑝−1 } ∪ [ 0, 𝑝 ]
2
Therefore E′ \ A ⊆ ( 0, 𝑝 ]
2
Since p∈ℕ is arbitrary, E′ \ A ⊆ ∩ (0, 𝑝 ] = ∅
p∈ℕ
1
Hence E′ = A = { 0 } ∪ { 𝑛 ∶ 𝑛 ∈ ℕ} (Ans).

49)Let S be a non-empty set of ℝ bounded below and 𝒎 = inf(S). If 𝒎 ∉ S, prove that 𝒎 is a


limit point of S.
Ans : → To show − ∀ε > 0, N′(𝑚, ε) ∩S ≠ ∅.
Take any ε > 0
Since 𝑚 = inf(S), ∃𝑥∈S such that 𝑥 < 𝑚 +ε
choose this 𝑥, since 𝑚 is a lower bound of S and 𝑚 ∉ S, then 𝑥 > 𝑚.
So, 𝑚 < 𝑥 < 𝑚 + ε ⇒ 𝑥∈(𝑚, 𝑚+ ε) ⊆ N′(𝑚,ε)
Therefore 𝑥 ∈ S and 𝑥∈N′(𝑚, ε) 𝑥
| |
⇒ 𝑥∈N′(𝑚,ε) ∩S 𝑚 𝑚+ε
Thus N′(𝑚,ε) ∩S ≠ ∅
As required.
50)Let S be a non-empty set of ℝ bounded above and M = SupS. If M ∉ S, prove that M is a limit
point of S. [C.U.04,12,]
Ans : → Let as an exercise.

51)Prove that the set of all rational numbers is not a closed set. [C.U.-13].
Ans : → Let, if possible ℚ be closed in ℝ.
Then ℚc is open in ℝ.
We know that √2 ∈ ℚc
So, ∃ε > 0, N(√2, ε) ⊆ ℚc
ℚc does not contain any rational number, but N(√2, ε) = (√2 − ε,√2 + ε), contains infinitely
many rationals.
A contradiction.
∴ ℚ is not closed in ℝ. i.e., the set of all rational numbers is not a closed set.

52)Define a closed set in ℝ in terms of its derived Set. [C.U. – 06].


Ans : → Let S ⊆ ℝ.The set S is called closed in ℝ if S contains all its limit points i.e, the derived
set S′⊆ S.

Example: -

(1) S = any finite set. Then S′ = ∅ ⊆ S. ∴ S is a closed Set.


1
(2) S = { 𝑛 ∶ 𝑛ϵℕ }. Then S′ = { 0 } ⊈ S. So S is not closed.

𝟏
53)Prove or disprove : ⋂∞
𝒏=𝟏(𝟎, 𝒏) is a closed set. [C.U.- 2019]

1
Answer : - To show ⋂∞
𝑛=1(0, 𝑛) = ∅

1
If possible, we let ⋂∞
𝑛=1(0, 𝑛) ≠ ∅

1
Then at least one element ∈ ⋂∞
𝑛=1(0, 𝑛)

1
⇒ ∀ 𝑛 ∈ ℕ, 0 < 𝑎 < 𝑛

But we know that, for any 𝑎 > 0, ∃𝑛 ∈ ℕ Such that 𝑛. 𝑎 > 1


(by Archimedean property)
A contradiction.
1
So ⋂∞
𝑛=1(0, 𝑛) = ∅ which is a closed Set.

54)Give an example of a set of real numbers which has exactly 3 limit points and the set is
closed. [C.U.-19]
1 1
Ans : → Let S = {(−1)𝑛 + 𝑛 : 𝑛∈ℕ } ∪ { 𝑛 : 𝑛∈ℕ } ∪ {−1}
We know that for any two non-empty sets A & B of ℝ, (A∪B)′ = A′∪B′
1 1
So, S′ = {(−1)𝑛 + 𝑛 : 𝑛∈ℕ }′ ∪ { 𝑛 : 𝑛∈ℕ }′ ∪ {−1}′

= {−1,1} ∪ {0} ∪ ∅ = {−1, 0,1}


Therefore S has exactly 3 limit points which are −1,0,1 and S′ ⊆ S.
Here S is closed.
55)The union of a finite number of closed sets in ℝ is a closed set.
Proof:
Let F1 , F2 ,-----, F𝑛 be closed sets in ℝ.
Let F =⋃𝑛𝑖=1 F𝑖
To show : F is closed in ℝ, i.e., F′ ⊆ F
Since each F𝑖 is closed, then F𝑖′ ⊆ F𝑖 for 𝑖 =1,2,-----,𝑛. ------ (1)
Now F′ = (⋃𝑛𝑖=1 F𝑖 )′ =⋃𝑛𝑖=1 F𝑖′ [ ∵ (A∪B)′ = A′∪B′ for any sets A, B ⊆ ℝ ]
From (i) F𝑖′ ⊆ F𝑖 ⊆ F, for all 𝑖 = 1,2,----, 𝑛
Then F′ = ⋃𝑛𝑖=1 F𝑖′ ⊆ F,
As required.
56)The intersection of a finite number of closed sets in ℝ is a closed set.

Proof :

Let F1 , F2 ,-----, F𝑛 be closed sets in ℝ


Let F = ⋂𝑛𝑖=1 F𝑖
To show : F is closed in ℝ, i.e., F′ ⊆ F
Since each F𝑖 is closed, then F𝑖′ ⊆ Fi for i =1,2,-----,𝑛 ------ (i)
Now, F′ = (⋂𝑛𝑖=1 F𝑖 )′ ⊆ ⋂𝑛𝑖=1 F𝑖′
Also F′ ⊆ ⋂𝑛𝑖=1 F𝑖′ ⊆ F𝑖′ ⊆ F𝑖 for 𝑖 = 1,2,-----,𝑛 [from(i)]
⇒ F′ ⊆ ⋂𝑛𝑖=1 F𝑖 = F.
As required.
57)The intersection of an arbitrary collection of closed sets in ℝ is a closed set.
Proof: Let {F∝ : ∝ ∈ ∧, ∧ being the index set}, be a collection of closed sets in ℝ.
Then F∝′ ⊆ F∝ for each ∝ ∈ ∧ ------ (i)
Let F = ∩ F∝
∝∈∧

To show : F is closed in ℝ i.e., F′ ⊆ F


Now, F′ = ( ∩ F∝ )′ ⊆ ∩ F∝′
∝∈∧ ∝∈∧

Also F′ ⊆ ∩ F∝′ ⊆ F∝′ ⊆ F∝ for each ∝∈∧ [from (i)]


∝∈∧

⇒ F′ ⊆ ∩ F∝ = F
∝∈∧

As required.
58)Let G be an open set in ℝ. Then prove that the complement of G is a closed set in ℝ.

Proof : -
Given that G is open.
To show : Gc is closed. i.e., (Gc )′ ⊆ Gc i.e., ∀𝑥∈(Gc )′ ⇒ 𝑥∈Gc
If G = ∅. Then Gc = ∅c = ℝ is obviously closed in ℝ.
If G ≠ ∅. Then take any 𝑥 ∈ G
Since G is open, then 𝑥 is an interior point of G. G
| ( | ) |
i.e., ∃ε > 0, N (𝑥,ε) ⊆ G 𝑥−𝜀 𝑥 𝑥+𝜀
⇒ ∃ε > 0, N (𝑥,ε) ∩ Gc = ∅
⇒ 𝑥 is not a limit point of Gc
⇒ 𝑥 ∉ (Gc )′
Therefore 𝑥 ∈ G ⇒ 𝑥 ∉ (Gc )′
Contrapositively, 𝑥∈(Gc )′ ⇒ 𝑥 ∉ G
i.e., 𝑥 ∈(Gc )′ ⇒ 𝑥 ∈ Gc
Since 𝑥 is arbitrary. ∴ (Gc )′ ⊆ Gc
So, Gc is closed in ℝ. (proved)
59)Let F be a closed set in ℝ. Then prove that the complement of F is an open set in ℝ.
Proof :-
Given that F is closed i.e., F′ ⊆ F
To show : F c is open i.e., ∀𝑥∈F c , ∃ε> 0 such that N(𝑥,ε) ⊆ F c .
If F = ℝ, then F c = ℝc = ∅ is open obviously. If F is a proper subset of ℝ. Then F c ≠ ∅.
Take any 𝑥 ∈ F c i.e., 𝑥 ∉ F ⇒ 𝑥 ∉ F′, since F′ ⊆ F.
So 𝑥 is not a limit point of F.
Then ∃ε > 0, N′(𝑥,ε) ∩F = ∅.
i.e., ∃ε > 0, N(𝑥,ε) ∩F = ∅. since 𝑥 ∉ F
⇒ N(𝑥,ε) ⊆ F c
So, for any 𝑥 ∈ F c , ∃ε > 0 such that N(𝑥,ε) ⊆ F c
∴ F c is open in ℝ (proved)
60)Prove that the complement of an open set is closed. Hence show that ℕ is closed. Also show
that for any 𝒙 > 𝟎, the set {𝒏𝒙 ∶ 𝒏 ∈ ℕ } is a closed set.
Answer :- 1st part : See Q. No (58)
2nd part : To show : ℕ is closed.
i.e., ℕ𝑐 is open.
Now, ℕ𝑐 = (−∞, 1) ∪ (1, 2) ∪ (2, 3) ∪ -----
We know that arbitrary union of open sets is an open set.
∴ ℕ𝑐 is open i.e., ℕ is closed (proved).
3rd part : Let S = {𝑛𝑥 ∶ 𝑛 ∈ ℕ} where 𝑥 > 0 .
To show : S is closed.
i.e., S c is open
Now, S c = (−∞, 𝑥) ∪ (𝑥, 2𝑥) ∪ (2𝑥, 3𝑥) ∪ -----
Also, S c is open i.e., S is closed (proved).
61)Let A ⊆ ℝ and G be an open set. If A′ denotes the set of all limit points of A then show
that G∩A′ = ∅, when G∩A = ∅ [C.U.-98,06]

Proof :-
Given that A is any set, G is any open Set and G∩A = ∅
To show : G∩A′ = ∅ G A
( )| |
Here G∩A = ∅
⇒ A ⊆ Gc
⇒ A′ ⊆ (Gc )′ ⊆ Gc , since G is open i.e., Gc is closed, So (Gc )′ ⊆ Gc
Since A′ ⊆ Gc ⇒ G ∩ A′ = ∅ (proved)
62)Examine whether the set {𝒙 ∈ ℝ ∶ 𝐜𝐨𝐬 𝟐𝒙 = 𝟎} is a closed Set. Prove that every
uncountable set of real numbers has a limit point. [C.U.-2019]
Answer : 1st part :- Let S = {𝑥 ∈ ℝ ∶ 𝑐𝑜𝑠2𝑥 = 0}

= {𝑥 ∈ ℝ ∶ 2𝑥 = (2𝑛 + 1), 𝑛 ∈ ℤ}
𝜋
= {(2𝑛 + 1) ∈ ℝ ∶ 𝑛 ∈ ℤ}
2
Here ∀𝑥 ∈ 𝑆, ∃ 𝜀 > 0 Such that N (𝑥, 𝜀) ∩ S = ∅. So S′ = ∅.
Therefore S′ ⊆ S, then S is closed.
2nd part :- To show : Every uncountable set of real numbers has a limit point.
If possible, we let there exists an uncountable set S of real numbers has no limit point.
∴ ∀𝑛 ∈ ℕ, 𝑆𝑛 = [−𝑛, 𝑛] ∩ 𝑆 is a finite set, Since 𝑆𝑛 is a bounded set which has no limit point.
Now, S = ⋃𝑛∈ℕ 𝑆𝑛 being a countable union of finite sets is countable.
A contradiction, Since S is uncountable.
So our assumption is incorrect.
∴ Every uncountable set has a limit point.
𝟏
63)Correct or Justify: {𝒙∈ℝ : cos 𝒙 = 0} is not a closed set. [C.U-12]
1
Ans: Let S = { 𝑥∈ℝ : cos 𝑥 = 0}
1
= {𝑥∈ℝ : = (2n+1)π/2, 𝑛∈ℤ }
𝑥
2
= { (2𝑛+1)𝜋 ∈ ℝ : 𝑛 ∈ ℤ }

Clearly, 0 is the limit point of S, but 0 ∉ S


So, S is not a closed set
The statement is true.
64)Define Adherent point of a Set.

Ans : → Let S ⊆ ℝ. Then 𝑥∈ℝ is called an adherent point of S if ∀ε > 0, N(𝑥, ε) ∩S ≠ ∅.

65)Define closure of a set with examples.


Ans : → Let S ⊆ ℝ. The set of all adherent points of S is called the closure of S and it is denoted
b𝑦 S̅.
i.e., S̅ = S∪S′, S′ is the derived set of S.
Example:
(1) Let S = ℕ. Then S̅ = S∪S′= ℕ∪∅ = ℕ, where S′= ∅
(2) Let S = ℚ; Then S̅ = S∪S′= ℚ ∪ ℝ = ℝ, where ℚ′ = ℝ
1 1
(3) Let S = { 𝑛 ∶ 𝑛 ∈ ℕ }. Then S̅ = S∪S′ = { 𝑛 ∶ 𝑛 ∈ ℕ } ∪ {0} where S′ = {0}.

66)Let S ⊆ ℝ. Then prove that 𝐒̅ is a closed set and it is the smallest closed set containing S.
_
Proof : 1st Part : - To show S is closed

i.e., ( S̅ )′ ⊆ S

Now ( S̅ )′ = (S∪S′)′ = S′∪(S′)′, since (A∪B)′ = A′∪B′

= S′, Since (S′)′ ⊆ S′

⊆ S∪S′ = S̅

This proves that S̅ is a closed Set .

2nd Part : -

Again S ⊆ S ∪ S′ = S̅ i.e., S̅ is a closed set containing S.

Let P be any closed set containing S.

Then S ⊆ P and S′⊆ P′ ⊆ P , since P is closed.


∴ S ⊆ P and S′ ⊆ P, combining these two we get

S ∪ S′ ⊆ P ⇒ S̅ ⊆ P.
_
Since P is arbitrary S is the smallest closed set containing S.

67)Define Boundary point of a set with examples.


Ans : → Let A⊆ ℝ and a∈ℝ. Then a is called a boundary point of A if ∀ ε > 0 ,N(a,ε) ∩A ≠ ∅
and N(a,ε) ∩ Ac ≠ ∅
The Boundary of A is the set of all boundary points of A.
̅ \ Ao
It is denoted by δ A = A
Example (i) A = (a, b) with a < b. Then a and b are only two boundary points of A.
(ii) If A is a finite set, then all points of A are boundary points of A.
68)Find the boundary of the following sets in ℝ .
𝟏
(i) {𝒏 : 𝒏∈ℕ}, (ii) [0,1] ∩ℚ, ℚ being the set of rationals
1
Ans : → (i) Let A = {𝑛 : 𝑛∈ℕ}. Then A′ = {0} and Ao = ∅
1
̅ \ Ao = (A ∪ A′)\ ∅ = A ∪ A′ = { : 𝑛∈ℕ } ∪ {0}
So δA = A 𝑛

ii) Let B = [0,1] ∩ ℚ


Since ℚ is dense in ℝ, then B is dense in [0,1]
_
So B′ = [0,1], B = B∪B′ = [0,1] and Bo = ∅.
_
∴ the boundary of B = δB = B \ Bo = [0, 1].
SETS IN ℝ

Exercise: -
1) Prove that the union of any arbitrary collection of open sets of real numbers is an
open set.

2) Find the derived set 𝑺′ of the following sets -


(−1)𝑚 1 1 1
(i) 𝑆 = { + 𝑛 : 𝑚, 𝑛 ∈ ℕ} (ii) S= {2m + 2n :m, n ∈ ℤ}
𝑚

3) Examine whether the following sets are -


(i) open or closed,
1 1
(ii) Neither open nor closed in ℝ → 𝑆 = {𝑥 ∈ ℝ: cos 𝑥 = 2} , 𝑇 = {𝑥 ∈ ℝ: sin 𝑥 = 0}

4) Let A be a non-empty bounded subset of ℝ. Show that {𝔁 ∈ ℝ: 𝒙 ≥ 𝒂 𝐟𝐨𝐫 𝐚𝐥𝐥 𝒂 ∈ 𝑨} is


closed subset of ℝ. [JAM-2012]

5) If K is a non-empty closed subset of ℝ, then show that the set {𝒙 + 𝒚: 𝒙 ∈ 𝑲, 𝒚 ∈ [𝟏, 𝟐]}
is closed in ℝ. [JAM-2014]

Multiple choice questions (MCQ)


1) Let 𝑨 = {(𝒙, 𝒚) ∈ ℝ𝟐 : |𝒙| + |𝒚 − 𝟏| < 𝟑}, 𝑩 = {(−𝟏)𝒏 . 𝒏: 𝒏 ∈ ℕ}
𝟏
𝑪 = {𝒙 ∈ ℝ: 𝒙 𝐬𝐢𝐧 𝒙 = 𝟎} ∪ {𝟎}. Which of the following is TRUE?

(a) B is closed, A and C are open, (b) B is open, A and C are closed,

(c) A, B, C are all open, (d) A is open, B and C are closed.

2) Any open subset of ℝ is -


(a) Union of countably many closed sets,
(b) Union of countably many open sets,
(c) Union of countably many disjoint open intervals,
(d) intersection of finitely many open intervals.

3) The set of all limit points of the set of all rational numbers is -
(a) Ø, (b) ℚ, (c) ℝ, (d) ℚ𝑐
𝟏 𝟏
4) The set {𝒏 𝐬𝐢𝐧 𝒏 ∶ 𝒏 ∈ ℕ} has – [NET – 2011(June)]
(a) One limit point and it is 0, (b) One limit point and it is 1,
(c) One limit point and it is (-1), (d) Three limit points and these are -1, 0 and 1.

𝟐
5) The set of all limit points of the set {𝒙+𝟏 : 𝒙 ∈ (−𝟏, 𝟏)} in ℝ is – [JAM – 2014]
(a) [1, ∞), (b) (1, ∞), (c) [-1, 1], (d) [-1, ∞)

6) Let X be a proper closed subset of [0,1]. Which of the following statements is always
TRUE? [TIFR – 2015]
(a) The set X is countable, (b) There exists 𝑥 ∈ 𝑋 such that 𝑋\{𝑥} is closed,

(c) The set X contains an open interval, (d) None of these.


𝒏 𝟏
7) Consider the following subsets of ℝ. 𝑬 = { : 𝒏 ∈ ℕ}, 𝑭 = { : 𝟎 ≤ 𝒙 < 𝟏}. Then
𝒏+𝟏 𝟏−𝒙

[JAM-2011]
(a) Both E and F are closed, (b) E is closed and F is not closed,
(c) E is not closed and F is closed, (d) Neither E nor F is closed.
𝒙
8) If 𝒀 = {𝟏+|𝒙| : 𝒙 ∈ ℝ}. Then the set of all limit points of Y is - [JAM - 2012]

(a) (-1,1), (b) (-1,1], (c) [0,1], (d) [-1, 1]

9) Let A and B be subsets of ℝ. Which of the following is NOT necessarily true?


[JAM – 2013]
(a) (𝐴 ∩ 𝐵)° ⊆ 𝐴° ∩ 𝐵 ° , (b) 𝐴° ∪ 𝐵 ° ⊆ (𝐴 ∪ 𝐵)° ,
(c) 𝐴 ∪ 𝐵 ⊆ (𝐴⋃𝐵), (d) 𝐴 ∩ 𝐵 ⊆ 𝐴 ∩ 𝐵

10) Let 𝑺 = {𝒙 ∈ ℝ: 𝒙𝟔 − 𝒙𝟓 ≤ 𝟏𝟎𝟎} and 𝑻 = {𝒙𝟐 − 𝟐𝒙: 𝒙 ∈ (𝟎, ∞)} . The set 𝑺 ∩ 𝑻 is
[JAM– 2014]
(a) closed and bounded in ℝ, (b) Closed but NOT bounded in ℝ,
(c) bounded but NOT closed in ℝ, (d) Neither closed nor bounded in ℝ.

11) Let 𝑺 ⊆ ℝ and 𝜹𝑺 denote the set of all points 𝒙 in ℝ such that every neighbourhood
of x contains some points of 𝑺 as well as some points of complement of 𝑺. Further, let
𝑺 denote the closure of 𝑺. Then which one of the following is FALSE?
[JAM – 2016]
(a) δ ℚ = ℝ, (b) δ (ℝ\T) = δT, 𝑇 ⊆ ℝ,
(c) δT = 𝑇 ∩ (ℝ\𝑇), 𝑇 ⊆ ℝ, (d) δ(𝑇 ∪ 𝑉) = δT ∪ δV; T, V ⊆ ℝ, T ∩ V ≠ ∅

12) The set {𝒙 ∈ ℝ: 𝒙 𝐬𝐢𝐧 𝒙 ≤ 𝟏, 𝒙 𝐜𝐨𝐬 𝒙 ≤ 𝟏} ⊆ ℝ is –


(a) a bounded closed set, (b) a bounded open set,
(c) an unbounded closed set, (d) an unbounded open set
13) Let 𝑨𝒊 : 𝒊 ∈ ℕ be a countable family of sets, then which of the following is NOT true?
(a) ⋃𝑖∈ℕ 𝐴𝑖 = ⋃𝑖∈ℕ 𝐴𝑖 , (b) (⋃𝑖∈ℕ 𝐴𝑖 )° = ⋃𝑖∈ℕ 𝐴𝑖 ° ,
(c) (⋃𝑖∈ℕ 𝐴𝑖 ) ∪ (⋃𝑖∈ℕ 𝐴𝑖 )′ = ⋃𝑖∈ℕ 𝐴𝑖 , (d) if 𝐴𝑖 is open for some i, then 𝐴𝑖 need not
be equal to 𝐼𝑁𝑇 (𝐴𝑖 ).
𝒏 √𝟐
14) Let S be the set of all limit points of the set { + : 𝒏 ∈ ℕ}. Let 𝑸+ be the set of all
√𝟐 𝒏

positive rational numbers. Then [JAM-2019]


(a) 𝑄+ ⊆ 𝑆, (b) 𝑆 ⊆ 𝑄+ , (c) 𝑆 ∩ (ℝ\𝑄+ ) ≠ ∅, (d) 𝑆 ∩ 𝑄+ ≠ ∅
𝟏
15) Let 𝑨 = {𝒙 ∈ ℝ: 𝟎 < 𝒙 ≤ 𝟏} and 𝑩 = {𝒏𝒏 : 𝒏 ∈ ℕ}. Then 𝑨\𝑩 is
(a) An open set (b) a closed set
(c) Both open and closed (d) Neither open nor closed
𝟏 𝟏
16) Number of limit points of the set 𝑺 = {𝟕 + 𝒏𝟐 : 𝒏 ∈ ℕ} ∪ {𝟕 − 𝒏 : 𝒏 ∈ ℕ} is
(a) 1 (b) 2 (c) 0 (d) infinite

Multiple Select Questions (MSQ)


1) Consider the intervals S = (0,2] and T = [1,3]. Let S° and T° be the sets of interior
points of S and T, respectively. Then the set of interior points of S\T is equal to
[JAM – 2019]
(a) S\T°, (b) S\T, (c) S°\T°, (d) S°\T

2) Let S be the set of all rational numbers in (0,1). Then which of the following
statements is / are true? – [JAM – 2017]
(a) S is closed subset of ℝ, (b) S is not a closed subset of ℝ,
(c) S is an open subset of ℝ, (d) Every 𝑥 ∈ (0,1) is a limit point of S.

𝟏 𝟏
3) The set 𝑺 = {𝟑𝒎 + 𝟕𝒏 ∶ 𝒎, 𝒏 ∈ ℕ}. Then which of the following(s) is(are) true?
[JAM-2016]
(a) S is closed, (b) S is not open,
(c) S is connected, (d) 0 is a limit point of S

You might also like